How to ensure when the derivative approaches zero, the function value approaches a constant?












0












$begingroup$


I asked a very similar question here. But now this is different. Suppose $f(t)$ is differentiable and $c$ is a finite constant, then the following statement looks correct, but in fact it is not:
begin{equation}
limlimits_{t to infty} f'(t) = 0 implies limlimits_{t to infty} f(t)=c
end{equation}

A counter-example is $f(t)=ln(t)$. Now the question is, what condition should be used for the above statement to be true?










share|cite|improve this question











$endgroup$

















    0












    $begingroup$


    I asked a very similar question here. But now this is different. Suppose $f(t)$ is differentiable and $c$ is a finite constant, then the following statement looks correct, but in fact it is not:
    begin{equation}
    limlimits_{t to infty} f'(t) = 0 implies limlimits_{t to infty} f(t)=c
    end{equation}

    A counter-example is $f(t)=ln(t)$. Now the question is, what condition should be used for the above statement to be true?










    share|cite|improve this question











    $endgroup$















      0












      0








      0


      1



      $begingroup$


      I asked a very similar question here. But now this is different. Suppose $f(t)$ is differentiable and $c$ is a finite constant, then the following statement looks correct, but in fact it is not:
      begin{equation}
      limlimits_{t to infty} f'(t) = 0 implies limlimits_{t to infty} f(t)=c
      end{equation}

      A counter-example is $f(t)=ln(t)$. Now the question is, what condition should be used for the above statement to be true?










      share|cite|improve this question











      $endgroup$




      I asked a very similar question here. But now this is different. Suppose $f(t)$ is differentiable and $c$ is a finite constant, then the following statement looks correct, but in fact it is not:
      begin{equation}
      limlimits_{t to infty} f'(t) = 0 implies limlimits_{t to infty} f(t)=c
      end{equation}

      A counter-example is $f(t)=ln(t)$. Now the question is, what condition should be used for the above statement to be true?







      real-analysis derivatives






      share|cite|improve this question















      share|cite|improve this question













      share|cite|improve this question




      share|cite|improve this question








      edited Dec 18 '18 at 17:12









      zhw.

      73.6k43175




      73.6k43175










      asked Dec 18 '18 at 15:59









      winstonwinston

      524218




      524218






















          1 Answer
          1






          active

          oldest

          votes


















          0












          $begingroup$

          An equivalent condition is that
          $$
          int_a^infty f'(t),dt
          $$

          converges as an improper Riemann integral for some $ainBbb R$. This happens for instance if $|f'(x)|le C,x^{-p}$ for some $Cge0$ and $p>1$.






          share|cite|improve this answer









          $endgroup$













          • $begingroup$
            Is there any proof of your result?
            $endgroup$
            – winston
            Dec 18 '18 at 16:39










          • $begingroup$
            $$f(x)=f(a)+int_a^xf'(t),dt,quad x>a.$$
            $endgroup$
            – Julián Aguirre
            Dec 18 '18 at 16:40










          • $begingroup$
            True if $f'$ is Riemann integrable on bounded intervals.
            $endgroup$
            – zhw.
            Dec 18 '18 at 17:14










          • $begingroup$
            Does it mean that $limlimits_{t to infty} f'(t)=0$ does not make any difference to the condition? Doesn't it somehow relax your condition in some way?
            $endgroup$
            – winston
            Dec 18 '18 at 20:21












          • $begingroup$
            If $lim_{xtoinfty}f'(x)$ exists, it must be $0$. But it may happen that the limit does not exist.
            $endgroup$
            – Julián Aguirre
            Dec 19 '18 at 8:28











          Your Answer





          StackExchange.ifUsing("editor", function () {
          return StackExchange.using("mathjaxEditing", function () {
          StackExchange.MarkdownEditor.creationCallbacks.add(function (editor, postfix) {
          StackExchange.mathjaxEditing.prepareWmdForMathJax(editor, postfix, [["$", "$"], ["\\(","\\)"]]);
          });
          });
          }, "mathjax-editing");

          StackExchange.ready(function() {
          var channelOptions = {
          tags: "".split(" "),
          id: "69"
          };
          initTagRenderer("".split(" "), "".split(" "), channelOptions);

          StackExchange.using("externalEditor", function() {
          // Have to fire editor after snippets, if snippets enabled
          if (StackExchange.settings.snippets.snippetsEnabled) {
          StackExchange.using("snippets", function() {
          createEditor();
          });
          }
          else {
          createEditor();
          }
          });

          function createEditor() {
          StackExchange.prepareEditor({
          heartbeatType: 'answer',
          autoActivateHeartbeat: false,
          convertImagesToLinks: true,
          noModals: true,
          showLowRepImageUploadWarning: true,
          reputationToPostImages: 10,
          bindNavPrevention: true,
          postfix: "",
          imageUploader: {
          brandingHtml: "Powered by u003ca class="icon-imgur-white" href="https://imgur.com/"u003eu003c/au003e",
          contentPolicyHtml: "User contributions licensed under u003ca href="https://creativecommons.org/licenses/by-sa/3.0/"u003ecc by-sa 3.0 with attribution requiredu003c/au003e u003ca href="https://stackoverflow.com/legal/content-policy"u003e(content policy)u003c/au003e",
          allowUrls: true
          },
          noCode: true, onDemand: true,
          discardSelector: ".discard-answer"
          ,immediatelyShowMarkdownHelp:true
          });


          }
          });














          draft saved

          draft discarded


















          StackExchange.ready(
          function () {
          StackExchange.openid.initPostLogin('.new-post-login', 'https%3a%2f%2fmath.stackexchange.com%2fquestions%2f3045325%2fhow-to-ensure-when-the-derivative-approaches-zero-the-function-value-approaches%23new-answer', 'question_page');
          }
          );

          Post as a guest















          Required, but never shown

























          1 Answer
          1






          active

          oldest

          votes








          1 Answer
          1






          active

          oldest

          votes









          active

          oldest

          votes






          active

          oldest

          votes









          0












          $begingroup$

          An equivalent condition is that
          $$
          int_a^infty f'(t),dt
          $$

          converges as an improper Riemann integral for some $ainBbb R$. This happens for instance if $|f'(x)|le C,x^{-p}$ for some $Cge0$ and $p>1$.






          share|cite|improve this answer









          $endgroup$













          • $begingroup$
            Is there any proof of your result?
            $endgroup$
            – winston
            Dec 18 '18 at 16:39










          • $begingroup$
            $$f(x)=f(a)+int_a^xf'(t),dt,quad x>a.$$
            $endgroup$
            – Julián Aguirre
            Dec 18 '18 at 16:40










          • $begingroup$
            True if $f'$ is Riemann integrable on bounded intervals.
            $endgroup$
            – zhw.
            Dec 18 '18 at 17:14










          • $begingroup$
            Does it mean that $limlimits_{t to infty} f'(t)=0$ does not make any difference to the condition? Doesn't it somehow relax your condition in some way?
            $endgroup$
            – winston
            Dec 18 '18 at 20:21












          • $begingroup$
            If $lim_{xtoinfty}f'(x)$ exists, it must be $0$. But it may happen that the limit does not exist.
            $endgroup$
            – Julián Aguirre
            Dec 19 '18 at 8:28
















          0












          $begingroup$

          An equivalent condition is that
          $$
          int_a^infty f'(t),dt
          $$

          converges as an improper Riemann integral for some $ainBbb R$. This happens for instance if $|f'(x)|le C,x^{-p}$ for some $Cge0$ and $p>1$.






          share|cite|improve this answer









          $endgroup$













          • $begingroup$
            Is there any proof of your result?
            $endgroup$
            – winston
            Dec 18 '18 at 16:39










          • $begingroup$
            $$f(x)=f(a)+int_a^xf'(t),dt,quad x>a.$$
            $endgroup$
            – Julián Aguirre
            Dec 18 '18 at 16:40










          • $begingroup$
            True if $f'$ is Riemann integrable on bounded intervals.
            $endgroup$
            – zhw.
            Dec 18 '18 at 17:14










          • $begingroup$
            Does it mean that $limlimits_{t to infty} f'(t)=0$ does not make any difference to the condition? Doesn't it somehow relax your condition in some way?
            $endgroup$
            – winston
            Dec 18 '18 at 20:21












          • $begingroup$
            If $lim_{xtoinfty}f'(x)$ exists, it must be $0$. But it may happen that the limit does not exist.
            $endgroup$
            – Julián Aguirre
            Dec 19 '18 at 8:28














          0












          0








          0





          $begingroup$

          An equivalent condition is that
          $$
          int_a^infty f'(t),dt
          $$

          converges as an improper Riemann integral for some $ainBbb R$. This happens for instance if $|f'(x)|le C,x^{-p}$ for some $Cge0$ and $p>1$.






          share|cite|improve this answer









          $endgroup$



          An equivalent condition is that
          $$
          int_a^infty f'(t),dt
          $$

          converges as an improper Riemann integral for some $ainBbb R$. This happens for instance if $|f'(x)|le C,x^{-p}$ for some $Cge0$ and $p>1$.







          share|cite|improve this answer












          share|cite|improve this answer



          share|cite|improve this answer










          answered Dec 18 '18 at 16:24









          Julián AguirreJulián Aguirre

          69.1k24096




          69.1k24096












          • $begingroup$
            Is there any proof of your result?
            $endgroup$
            – winston
            Dec 18 '18 at 16:39










          • $begingroup$
            $$f(x)=f(a)+int_a^xf'(t),dt,quad x>a.$$
            $endgroup$
            – Julián Aguirre
            Dec 18 '18 at 16:40










          • $begingroup$
            True if $f'$ is Riemann integrable on bounded intervals.
            $endgroup$
            – zhw.
            Dec 18 '18 at 17:14










          • $begingroup$
            Does it mean that $limlimits_{t to infty} f'(t)=0$ does not make any difference to the condition? Doesn't it somehow relax your condition in some way?
            $endgroup$
            – winston
            Dec 18 '18 at 20:21












          • $begingroup$
            If $lim_{xtoinfty}f'(x)$ exists, it must be $0$. But it may happen that the limit does not exist.
            $endgroup$
            – Julián Aguirre
            Dec 19 '18 at 8:28


















          • $begingroup$
            Is there any proof of your result?
            $endgroup$
            – winston
            Dec 18 '18 at 16:39










          • $begingroup$
            $$f(x)=f(a)+int_a^xf'(t),dt,quad x>a.$$
            $endgroup$
            – Julián Aguirre
            Dec 18 '18 at 16:40










          • $begingroup$
            True if $f'$ is Riemann integrable on bounded intervals.
            $endgroup$
            – zhw.
            Dec 18 '18 at 17:14










          • $begingroup$
            Does it mean that $limlimits_{t to infty} f'(t)=0$ does not make any difference to the condition? Doesn't it somehow relax your condition in some way?
            $endgroup$
            – winston
            Dec 18 '18 at 20:21












          • $begingroup$
            If $lim_{xtoinfty}f'(x)$ exists, it must be $0$. But it may happen that the limit does not exist.
            $endgroup$
            – Julián Aguirre
            Dec 19 '18 at 8:28
















          $begingroup$
          Is there any proof of your result?
          $endgroup$
          – winston
          Dec 18 '18 at 16:39




          $begingroup$
          Is there any proof of your result?
          $endgroup$
          – winston
          Dec 18 '18 at 16:39












          $begingroup$
          $$f(x)=f(a)+int_a^xf'(t),dt,quad x>a.$$
          $endgroup$
          – Julián Aguirre
          Dec 18 '18 at 16:40




          $begingroup$
          $$f(x)=f(a)+int_a^xf'(t),dt,quad x>a.$$
          $endgroup$
          – Julián Aguirre
          Dec 18 '18 at 16:40












          $begingroup$
          True if $f'$ is Riemann integrable on bounded intervals.
          $endgroup$
          – zhw.
          Dec 18 '18 at 17:14




          $begingroup$
          True if $f'$ is Riemann integrable on bounded intervals.
          $endgroup$
          – zhw.
          Dec 18 '18 at 17:14












          $begingroup$
          Does it mean that $limlimits_{t to infty} f'(t)=0$ does not make any difference to the condition? Doesn't it somehow relax your condition in some way?
          $endgroup$
          – winston
          Dec 18 '18 at 20:21






          $begingroup$
          Does it mean that $limlimits_{t to infty} f'(t)=0$ does not make any difference to the condition? Doesn't it somehow relax your condition in some way?
          $endgroup$
          – winston
          Dec 18 '18 at 20:21














          $begingroup$
          If $lim_{xtoinfty}f'(x)$ exists, it must be $0$. But it may happen that the limit does not exist.
          $endgroup$
          – Julián Aguirre
          Dec 19 '18 at 8:28




          $begingroup$
          If $lim_{xtoinfty}f'(x)$ exists, it must be $0$. But it may happen that the limit does not exist.
          $endgroup$
          – Julián Aguirre
          Dec 19 '18 at 8:28


















          draft saved

          draft discarded




















































          Thanks for contributing an answer to Mathematics Stack Exchange!


          • Please be sure to answer the question. Provide details and share your research!

          But avoid



          • Asking for help, clarification, or responding to other answers.

          • Making statements based on opinion; back them up with references or personal experience.


          Use MathJax to format equations. MathJax reference.


          To learn more, see our tips on writing great answers.




          draft saved


          draft discarded














          StackExchange.ready(
          function () {
          StackExchange.openid.initPostLogin('.new-post-login', 'https%3a%2f%2fmath.stackexchange.com%2fquestions%2f3045325%2fhow-to-ensure-when-the-derivative-approaches-zero-the-function-value-approaches%23new-answer', 'question_page');
          }
          );

          Post as a guest















          Required, but never shown





















































          Required, but never shown














          Required, but never shown












          Required, but never shown







          Required, but never shown

































          Required, but never shown














          Required, but never shown












          Required, but never shown







          Required, but never shown







          Popular posts from this blog

          How do I know what Microsoft account the skydrive app is syncing to?

          When does type information flow backwards in C++?

          Grease: Live!